LSAT and Law School Admissions Forum

Get expert LSAT preparation and law school admissions advice from PowerScore Test Preparation.

 Administrator
PowerScore Staff
  • PowerScore Staff
  • Posts: 8917
  • Joined: Feb 02, 2011
|
#27225
Complete Question Explanation

The correct answer choice is (E)

The main point of passage B is to highlight the failure of Monetarism as applied to Great Britain in the 1980s. The author did not imply that Thatcher did not understand the concept of monetarism (answer choice A). The author of passage B was certainly not focused on the theoretical value of Friedman’s theories (answer choice B). Answer choice C is contrary to the passage because of the word diverge, and we can’t be sure what Friedman considered in developing his approach (answer choice D).
 jmramon
  • Posts: 47
  • Joined: Jul 21, 2017
|
#40956
I got every question correct except the MP, so I'm trying to reconcile the following: How can answer A for #4 be correct when answer B for #1 is incorrect? Could someone please explain? What's the difference between something being "valuable in theory and not practice" (B on #1) and something "working in theory" (A on #4)? I think the person ahead of me kind of answered my question, but just to be sure--is the difference that of value versus plausibility?
 James Finch
PowerScore Staff
  • PowerScore Staff
  • Posts: 943
  • Joined: Sep 06, 2017
|
#40980
Hi JM,

There are a couple issues with (B). While Passage B notes that the practical results differed from the theoretical application of monetarism, nowhere does the author imply that monetarism is "very valuable" in theory, nor does "not quite as valuable" match the tone with which the author describes the British application of monetarism in the 1980's. (E) is explicitly stated in the first sentence of the last paragraph, and accords with the tone of the prior and subsequent description in the passage of the effects of monetarist policy on the British economy.

As to question 4, "works" is far less positive in tone than "very valuable," and connotes internal consistency rather than value. So (A) on question 4 may be true while (B) on question 1 is not.

Hope this clears things up!

Get the most out of your LSAT Prep Plus subscription.

Analyze and track your performance with our Testing and Analytics Package.